LSAT and Law School Admissions Forum

Get expert LSAT preparation and law school admissions advice from PowerScore Test Preparation.

 Administrator
PowerScore Staff
  • PowerScore Staff
  • Posts: 8916
  • Joined: Feb 02, 2011
|
#91389
Complete Question Explanation

The correct answer choice is (E).

Answer choice (A):

Answer choice (B):

Answer choice (C):

Answer choice (D):

Answer choice (E): This is the correct answer choice.

This explanation is still in progress. Please post any questions below!
 menkenj
  • Posts: 116
  • Joined: Dec 02, 2020
|
#90678
I personally found this question to be the most difficult of the entire RC section.

The question stem is asking us to strengthen the argument that increasing the number of specifies will likely increase the number of species needing protection. It has a causal feel to it.

A. This answer does nothing to support the relationship between the number of species and the number needing protection. The agreements to protect endangered species comes after, so is irrelevant to the relationship in question.

B. Irrelevant. Having more avian species vs. mammalian specifies tells us nothing about the number of species needing protection. Perhaps in this scenario it might be more likely that more of those new mammalian speciies are endangered than the new birds and could actually weaken the argument. We don't know so it's wrong.

C. The weight given to considerations in determining which species to protect is irrelevant to the relationship we are trying to strengthen. And this may even weaken the argument since if economic considerations take precedence, then perhaps it would result in fewer species being protected. Protecting endangered species requires money and there is likely a limit on how much can be spent on protecting endangered species.

D. Very attractive answer choice. If advancements in the DNA technique favors the splitters then it's likely that we will see an increase in the number of species. However, this answer choice doesn't bring it home and relate this to the author's conclusion that this would result in a greater number of species needing protection. So ultimately this does not strengthen the relationship in the same way as E.

E. Correct Answer. This answer is saying that splitters are less likely to challenge a species classification if they are not endangered. The answer choice implies that splitters are more likely to challenge a species classification when they ARE endangered. When lumpers lump together species which are not endangered, splitters say, "cool, no big deal, they aren't endangered so I'm not gonna fight you," and those species get lumped together (meaning fewer species that are not endangered). When lumpers lump together species that are endangered, splitters challenge the classification and fight for them to be separate species. Even if lumpers only win a fraction of these challenges, it would still result in a greater number of species needing protection. This answer strengthens the conclusion by reinforcing the relationship between # species and #species needing protection and is correct.


Does this analysis work well? Has anyone else developed an analysis for E? I'm curious to see if there is another explanation.
User avatar
 takeitslow_13
  • Posts: 1
  • Joined: Sep 30, 2021
|
#90886
Hi menkenj,

This was definitely a tricky question to process!

My take on (E) would be as follows:

The last sentence of the passage gives us a causal chain that looks like this
Increase number of species → Increase number of species needing protection

(E) gives us a conditional statement that says
No endangered populations involved → PSC LESS likely to contest an established species classification

The Contra-positive of (E) would sound like this
PSC NOT LESS LIKELY to contest a classification (ie. likely to lead to INCREASED number of species) → SOME endangered populations involved (species needing protection)

PSC NOT LESS LIKELY to contest a classification: This could be understood as PSC proponents being just as or more likely to contest a classification which would mean an INCREASED number of species

SOME endangered populations involved: This would mean there are at least SOME endangered species involved that need protection.

So, overall the contrapositive of (E) gives us a conditional claim that if we have MORE species (via PSC contesting) then it is likely that there will be endangered species that need protection. This seems to provide the most support for the last sentence in the passage.

Look forward to hearing any feedback on this!
User avatar
 German.Steel
  • Posts: 55
  • Joined: Jun 12, 2021
|
#91004
This question struck me as the most difficult question of the entire LSAT 91, and one of the more challenging RC questions of the "modern" LSAT (tests 70+ or so), so I can't resist giving my take on it.

First of all, it's crucial to recognize that this is a straight-up LR question transplanted to RC. As such, your LR intuitions should be kicking in, and these intuitions should lead you to note that on tricky "which would most strengthen" LR questions, it's not enough to simply find an answer that could ostensibly strengthen the argument; you have to choose the answer that most strengthens, and sometimes that means dispatching an answer choice that does, in fact, seem to strengthen the argument.

With that in mind, let's move on to the answer choices. (A), (B), and (C) are clearly dumb, so let's get down to (D) and (E):

(D) struck me as a definite strengthener upon first read. If DNA-DNA advances are made, then that will probably favor the "splitters," which means more species are likely to be recognized. And even though this answer doesn't directly address increasing the number of endangered species, which is a definite drawback, any mathematician/statistician worth his salt would tell you that increasing the absolute number of species, given each species has a nonzero chance of being endangered, will likely increase the number of species considered "endangered."

But if you step back and think about it in the context of the author's statement, you'll realize that the author's statement is CONDITIONAL upon an increase in the number of species, so all this answer choice really does is make it MORE LIKELY that the number of species will increase, but doesn't speak to the number of endangered species increasing. You can't strength an argument that is conditional up a certain thing happening by saying "that certain thing is more likely to happen." So all that nonsense about mathematician/statisticians is just one big unsubstantiated assumption that doesn't help the argument within the context of this answer choice.

Not to mention the weak language ("IF advances are made"..."will PROBABLY favor the splitters..."), plus the lack of a specific mention of endangered species, left me feeling quite disturbed by this answer, so I moved onto (E) with an open mind.

(E) definitely does the trick, but you have to understand how to properly manipulate causal statements to get there. It wasn't immediately clear to me what effect this "less likely to contest" statement had, so I flipped it: splitters are more likely to contest a species classification if some of the species in question are endangered. And that, as opposed to (D) with its weak language and non-mention of endangered species, is a slam dunk strengthener: species that are endangered are more likely to be split into further species if splitters have their way (which they presumably would if species are increasing), thus increasing the number of endangered species overall.

One final note: I think another tricky aspect of this question is reading precisely that we're trying to strengthen the notion that an increase in species generally will lead to an increase in the number of species classified as endangered, not an increase in the overall number of individual organisms being classified as endangered. That misunderstanding could lead you discard (E), because surely the number of organisms recognized as endangered wouldn't necessarily increase just because you split an endangered species into several different species. But alas, that's not the point we're trying to strengthen.

Lots of trickery with this question.
 Adam Tyson
PowerScore Staff
  • PowerScore Staff
  • Posts: 5153
  • Joined: Apr 14, 2011
|
#91776
There has already been a lot of great analysis done here by several students, and I have little to add to it! I agree that this is a VERY hard question, and E is the best answer for the reasons given by all of you (excellent work!)

I would like to clarify what's wrong with answer D, though. We are supposed to be strengthening the claim that increasing the number of species would also increase the number that need protection. Answer D does nothing at all to address that mathematical idea! So what if advances in technology assist the phylogenic theory? That wouldn't mean the phylogenic theory was any more correct, just that they would have better tools at their disposal. It has nothing to do with what the consequences of accepting their theory would mean!

Answer E indicates that any increases under that theory would be more likely to involve endangered species, perhaps splitting one endangered group into two endangered groups. It's ultimately just a numbers and percentages question, and if it's true, then the claim at the end of the passage is supported, even if not entirely proven.

Again, good work by you all, menkenj, takeitslow_13, and German.Steel!
User avatar
 Becca1924
  • Posts: 11
  • Joined: Dec 06, 2021
|
#92667
Hi! There are excellent explanations for the answer choices on this question, but I am having a hard time understanding the question itself. Specifically, what exactly is the argument in the sentence referenced? That is the first step for me in figuring out how to strengthen it.
 Robert Carroll
PowerScore Staff
  • PowerScore Staff
  • Posts: 1787
  • Joined: Dec 06, 2013
|
#92675
Becca,

There really is no argument. The author made an assertion in the last sentence without any evidence for it. So this question is asking us to take that assertion as the conclusion of an argument (an argument that, as of yet, has no premises). Then we strengthen that conclusion.

This isn't a very common thing - usually the assertion you're strengthening for a Strengthen question is the conclusion of an argument that already has some evidence for it - but it's not unique - there's an LR question similar to this, with the stimulus being an assertion unsupported by any evidence: viewtopic.php?f=576&t=11309 But note that, in any case, this is a Strengthen question, which means you're helping the point in the last sentence. No different from any other Strengthen question in that respect!

Robert Carroll
User avatar
 Becca1924
  • Posts: 11
  • Joined: Dec 06, 2021
|
#92762
Okay, thanks! How strange.
User avatar
 sabinarose99
  • Posts: 7
  • Joined: Apr 11, 2022
|
#95764
Why is A wrong? Isn't the last sentence saying increase species, then you get more needing protection? And A says it's becoming harder to protect endangered species. So if you split up all the species, wouldn't more be endangered? Or did I just make that assumption?
User avatar
 katehos
PowerScore Staff
  • PowerScore Staff
  • Posts: 184
  • Joined: Mar 31, 2022
|
#95804
Hi sabinarose99, thanks for your question!

You're correct that the last sentence asserts that increasing the number of species would increase the necessity for protection!

In terms of why (A) is incorrect, whether or not international agreements to protect endangered species are more difficult to enact is irrelevant to the assertion in question. Consider the specific relationship we're looking to support: a greater number of species likely means a greater number of species need protection. Try to ask if international agreements being difficult (or easy) impacts the actual number of species in existence? It doesn't! Whether or not it is difficult to enact an international agreement does not affect the number of species nor the number of species in need of protection (though it certainly might affect solutions to the issues endangered species face; unfortunately, that's not the question we're being asked).

Answer choice (E), on the other hand, helps illustrate how the different positions on species might increase the number of endangered species and thus the number of species that need protection. As some others have explained above, (E) provides us with the knowledge that 'splitters' are less likely to split a species into new species if none of them are endangered (as mentioned by another user on the forum, this can be translated to mean splitters are more likely to split a species into new species if some of them are endangered). Knowing this, we can now support the last sentence because we know that splitters are more likely to create new species, especially when members of the population are endangered!

I hope this makes sense :)
Kate

Get the most out of your LSAT Prep Plus subscription.

Analyze and track your performance with our Testing and Analytics Package.